2(w-9)=5w+6+w i need help on this question?

Answers

Answer 1

Answer:

w=-6

Step-by-step explanation:

2(w-9)=5w+6+w

multiply 2 to the (w-9), also add the 5w to the other w

2w-18=6w+6

subtract 2w to 6w

-18=4w+6

subtract 6 to -18

-24=4w

divide 4 to -24

THE ANSWER IS -6


Related Questions

Pls help I don’t understand

Answers

Answer

B) a bisector
Answer
B) bisector!!!

Find the midpoint of (-4,6) and (10,-10)

Answers

Answer:

(3,-2)

Step-by-step explanation:

10+4=14; 14/2=7

-4+7=3

x=3

6+10=16; 16/2=8

6-8=-2

y=-2

Answer:

(3,-8)

Step-by-step explanation:

m=x1+x2/2,y1+y2/2

m--4+10/2,-10-6/2

m=6/2,-16/2

m=3,-8

find the first five terms of each sequence an = -2(n-7)

Answers

Answer:

when n=1

= -2(1-7) = 12

when n=2

= -2(2-7) = 10

when n=3

= -2(3-7) = 8

when n=4

= -2(4-7) = 6

when n=5

= -2(5-7) = 4

Step-by-step explanation:

Give all answers in standard form.
(3 x 10”) x (2 x 10)

Answers

Answer:

[tex]6 \times {10}^{2} [/tex]

Step-by-step explanation:

[tex](3 \times 10) \times (2 \times 10) \\ 30 \times 20 \\ = 600 \\ = 6 \times {10}^{2} [/tex]

order from least to greatest: -0.8, 2/5, 0.825, -3/5

Answers

-0.8, -3/5, 2/5, 0.825

I need help! With this problem

Answers

Answer:

1.) 5^7     2.) 4^a+b     3.) a^c+d

Step-by-step explanation:

If the exponents are number, you can add the numbers together. Do not add the base numbers. ( Ex: 3 + 4 = 7 )(problem 1)

If the exponents are variables, indicate that you are adding the variables by putting an addition sign between the letters.

Evaluate 35.2 - 62.71

Answers

Answer:

-27.51

Step-by-step explanation:

subtract the two numbers

= -2751/100 = -27.51

Which of the following is equivalent to 125%


7\4

0.125

5/4

1.025

Answers

[tex]\huge\boxed{\boxed{\bold{\frac{5}{4}}}}[/tex]

[tex]\hrulefill[/tex]

Let's start by finding [tex]125\%[/tex] in decimal form. To do this, just divide the percentage by [tex]100[/tex]:

[tex]125\div100=1.25[/tex]

This doesn't match any of our choices, so we'll try fraction form.

A percentage in fraction form is just the percentage over a denominator of [tex]100[/tex].

[tex]\frac{125}{100}[/tex]

Now we divide both the numerator and denominator by their greatest common factor, which is [tex]25[/tex]:

[tex]\frac{125\div25}{100\div25}=\large\boxed{\frac{5}{4}}[/tex]

BRAINLIESTT I PROMISEEE

Answers

Answer:

-56

Step-by-step explanation:

t=-9 u=7

(-9)(7) + (7) =

-63 + (7) =

-56

Answer:

-56

Step-by-step explanation:

-9(7)=-63

-63+7=-56

If Kim's age is twice of her sister. When you add Kim's age to her sister age, you get 36. How old is each sister? A) Write an equation that represents the situation. Explain any variable used. B)solve the equation from part (a)show your work. State your solution as a complete sentence. Thank you

Answers

Answer:

Let's kim's sisters age be 2x and kim's age be x.

★ When we add kim's age to her sister age, they equals to 36.

So, the equation becomes -

[tex] \\ \implies \sf \: x + 2x = 36 \\ \\ \\ \implies \sf \: 3x = 36 \\ \\ \\ \sf \underline{Now, \: \: solve \: \: for \: x \: \: - } \\ \\ \\ \implies\sf \: x = \frac{36}{3} \\ \\ \\ \implies \large{ \boxed{ \sf{ x = 12}}} \\ [/tex]

Kim's sisters age = 12 years.Kim's age = 2x = 2 × 12 = 24 years.

Ryan has a collection of 216 baseball cards. He lets his brother have 1/3 of his collection and takes 27 cards to school to give his friends. How many
baseball cards remain in Ryan's collection?

Answers

Answer:

117

Step-by-step explanation:

216/3=72

216-72=144

144-26=117

Answer: 117

Step-by-step explanation:

First you do 216 divide by 3 to know how many cards he gave his brother. So 216/3 is equal to 72. Then you add 72 to 27 to know the total amount of cards he loses, which is 99. Now you do 216-99 to get 117.

117 baseball cards remain in Ryan's collection.

HELP ASAP:Which table represents a function?

Answers

The answer is the first one

an input cant have different outputs

Given: KM bisects JKL
Prove: m MKL= 1/2m JKL

please explain

Answers

Answer:

Step-by-step explanation:

Given :

KM bisects JKL

To Prove :

m∠MKL = [tex]\frac{1}{2}\angle \text{JKL}[/tex]

                    Statements                                      Reasons

1). KM bisects ∠JKL                                 1). Given

2). m∠JKM = m∠MKL                              2). Definition of congruent angles

3). m∠JKM + m∠MKL = m∠JKL              3). Angle addition postulate

4). m∠MKL + m∠MKL = m∠JKL              4). Substitution property of equality

5). 2m∠MKL = m∠JKL                            5). Distributive property

6). m∠MKL = [tex]\frac{1}{2}\angle \text{JKL}[/tex]                               6). Division property of equality

help help help please :)​

Answers

-5 ≤ 1 - 2x < 9

Let's first break this down into two different inequalities.

1 - 2x ≥ -5

Subtract 1 from both sides.

-2x ≥ -6

Divide both sides by -2

x ≤ 3

Now, let's do the second inequality.

1 - 2x < 9

Subtract 1 from both sides.

-2x < 8

Divide both sides by -2

x > 4

let's combine each inequality to get our reduced compound inequality.

-4 < x ≤ 3

To plot this, put a hollow point at -4. A hollow point looks something like the letter o.

And put the second point at 3. This time, fill in the point so it looks whole.

Now, connect your points.

It should look something like this:

3x - y = 2
and
2x + y = 4​

Answers

[tex]\frac{3}{1}[/tex]Answer:

Step-by-step explanation:

3x-y = 2

2x+y =4

put them into slope intercept form

y=3x-2

y=-2x+4

Now, you can either graph it or use substitution

3x-2 = -2x+4

+2x +2x

5x -2 = 4

+2 +2

5x = 6

5x/5 = 6/5

x = 6/5

3x -2 = y

3(6/5) -2 = y

3 x 6 / 1 x 5 = 18/5

3 3/5 -2 = 1 3/5

Let me know if it's wrong.

Hope this helps

1 USD is equal to 1.03069 Canadian dollars Mitchell travels from the US to Canada, where he exchanges 150 US dollars for Canadian dollars. He then spends 20 Canadian dollars, returns to the US, and exchanges the remaining money back to US dollars. How many US dollars does Mitchell have remaining?

Answers

Answer:

130$

Step-by-step explanation:

Answer:

Mitchell will have 130.66 USD remaining.                                                              

                                                                                                                                                                                     

{{3x + 6}/2- 4}/2 + 12/2=10
the solution for x is:
A). x = 4
B). x = -4
C). x = -2
D). x = 2

Answers

Answer:

The answer is D). x=2

Step-by-step explanation:

If 4 tickets to a show cost$9.00, find the cost of 14 tickets.

Answers

Answer:

Step-by-step explanation:

Answer:

31.50 for 14 tickets

Step-by-step explanation:

We can use ratios to solve

4 tickets          14 tickets

--------------   = --------------------

9 dollars          x dollars

Using cross products

4x = 9*14

4x =126

Divide each side by 4

4x/4 = 126/4

x =31.50

How many calories will Jacob burn in 1 minute while roller-skating????

Answers

Answer:

the answer is five

Step-by-step explanation:

first think of it like 40 over 200 then simplified down. which will get you 1/5 ,therefore five is the answer.


Help please. I don’t understand and my teacher didn’t bother to teach us any of this and my brain hurts!!!!

Answers

Answer:

  23.  42 m^2

  24.  144 ft^2

  25.  68 cm^2

Step-by-step explanation:

The teacher should not need to spend any time teaching this. The area of the total figure is the sum of the areas of its parts. You already know that. And, you already know how to compute the areas of triangles and rectangles.

__

You may recall that the area of a triangle is ...

  A = 1/2bh

and the area of a rectangle with the same base and height is ...

  A = bh

So, a triangle with base b and height h has the same area as a rectangle with the same base (b) and half the height (h/2). Recognizing this makes these problems really simple.

__

23. The base of the rectangle and triangle is 7 m. The height of the triangle is 4 m, so its area is equivalent to adding 4/2 = 2 m to the 4 m height of the rectangle. Then the total area is ...

  A = (7 m)(4 m +(1/2)(4 m)) = (7 m)(6 m) = 42 m^2

__

24. Since there are two identical triangles of height 5, their total area is equal to that of a rectangle of height 5.

  A = (12 ft)(7 ft +5 ft) = 144 ft^2

__

25. The 5 cm height of the triangle makes it have an area equivalent to a rectangle 5/2 cm high.

  A = (8 cm)(6 cm +5/2 cm) = 68 cm^2

Note: we have read the fuzzy numbers as 8 cm wide, 6 cm high for the rectangle, and 5 cm high for the triangle. If your figure is different, please use the appropriate numbers in the calculation.

In a bag there are 3 red marbles, 2 yellow marbles, and 1 blue marble. What are the odds of a yellow marble being selected on the first draw? CHOICES ------> ( 4:2 1:5. 3:3. 2:4) SOS

Answers

Answer:

1:3

Step-by-step explanation:

To find the probability of selecting a certain type of item from a collection, simply divide the number of items of that type by the total number of items.

There are 3 + 2 + 1  = 6 marbles in total. There are 2 yellow marbles, so the probability of choosing a yellow marble on the first draw is 2/6. This can be further simplified to 1:3.

Answer:

i would say 2:4

Step-by-step explanation:

think about them as basketball teams

The yellow team has two players

the mix team has 4 players

so the game is 2 players on 4 players

A car covers a distance of 14.75km in 20 minutes. In how many seconds, will it cover a distance of 73.75km

Answers

Answer:

6000 seconds.

Step-by-step explanation:

Given:

A car covers a distance of 14.75km in 20 minutes.

find:

In how many seconds, will it cover a distance of 73.75km

solution:

the idea is ratio and proportion to get the time based on 73.75 km from 14.75km./20min. in seconds.

 14.75 km.    =  73.75 km    --------> cross multiply

   20 min.              t

14.75 (t) = 73.75 (20)

14.75 (t) = 1475

t =    1475  

      14.75

t = 100 min.  -----------> convert min to seconds.

100 min. x   60 secs.      =          6000 seconds

                      1 min.

therefore,

the time in seconds to travel a distance of 73.75 km is 6000 seconds.

6. Evaluate the function at each specified value of the independent variable and simplify. (If an answer is undefined, enter UNDEFINED.) 7. Evaluate the function at each specified value of the independent variable and simplify. (If an answer is undefined, enter UNDEFINED.)

Answers

Complete Question

The complete question is shown on the first uploaded image

Answer:

6a

    [tex]V(3) =  113.112[/tex]

6b

     [tex]V(\frac{3}{2})  = 14.139  [/tex]

6c

   [tex]V(2r) =  33.514r^3[/tex]

7a

  [tex] h(5) = 0 [/tex]

7b

    [tex] h(t) = -5.76 [/tex]

7c

[tex] h(x+5) = x(x+5)[/tex]

Step-by-step explanation:

Considering the question 6

The function given is [tex]V(r) = \frac{4}{3} \pi r^3[/tex]

For  V(3) we have  

        [tex]V(3) =  \frac{4}{3} \pi 3^3[/tex]

       [tex]V(3) =  \frac{4}{3} * 3.142 * 3^3[/tex]

       [tex]V(3) =  113.112[/tex]

For  [tex]V(\frac{3}{2})[/tex]

        [tex]V(\frac{3}{2})  =  \frac{4}{3} *  \pi *   (\frac{3}{2})^3 [/tex]

        [tex]V(\frac{3}{2})  =  \frac{4}{3} * 3.142 *   (\frac{3}{2})^3 [/tex]

        [tex]V(\frac{3}{2})  = 14.139  [/tex]

For   V(2r)

            [tex]V(2r) =  \frac{4}{3} \pi (2r)^3[/tex]

           [tex]V(2r) =  \frac{4}{3} * 3.142*8r^3[/tex]

           [tex]V(2r) =  33.514r^3[/tex]

Considering the question 7

     The function given is  [tex] h(t) = t^2 - 5t [/tex]

For  h(5)

          [tex] h(5) = 5^2 - 5(5) [/tex]    

           [tex] h(5) = 0 [/tex]  

For  h(1.8)

            [tex] h(t) = 1.8^2 - 5(1.8) [/tex]

            [tex] h(t) = -5.76 [/tex]

For  h(x+ 5)

              [tex] h(x+5) = (x+5)^2 - 5(x+5) [/tex]

               [tex] h(x+5) = x^2 + 25+10x-5x-25[/tex]

             [tex] h(x+5) = x(x+5)[/tex]

what type of angle is <CEB? ​

Answers

Answer:

obtuse angle

Step-by-step explanation:

m(<CEA)=m(<BED)=88 (vertically opposite angles)

So m(<CEB)+m(<AED)=360-(88+88)=182

m(<CEB)=m(<AED)= 182÷2=92

Assuming the angle below is drawn to scale, which of the following could be the measure of 2XYZ? A. 120° B. 90 C. 180° O D. 30°​

Answers

Answer:

Option (A)

Step-by-step explanation:

Given angle in the picture is neither a right angle (equal to 90°) nor obtuse angle (greater than 90°).

It's an acute angle which measures less than 90°.

Therefore, possible measure of the given acute ∠XYZ = 30°

Option (A) will be the answer.

Each day for 6 days, the cooks at a school prepared
298 meals for lunch. How many meals did the cooks
prepare in the 6 days?
A
1,748 meals
Nath
B
1,788 meals
2.288 meals
D
596 meals

Answers

Answer:

B

Step-by-step explanation:

Because 298 time 6 is 1,788 :)

Which equation is the quadratic regression equation for the data shown in the table?

Answers

Answer: y=0.392x2 - 5.583x +21.167

Step-by-step explanation:

Please help it’s due on Monday!

Answers

The rounded answer is 9.1 hours. The non-rounded answer is 9.11428571429 hours.

First convert all the fractions to decimals, add all the decimals, and then divide by the number of countries to get your answer.

Add in your average use too for the problem.

600 ÷ 300 hey answers

Answers

Answer:

2

Step-by-step explanation:

I hope this helps you :)

Answer:

2

Step-by-step explanation:

Anything divided by 2 is half of itself. So 600 ÷ 300 would technically be 300. But in division we count the amount of times a number can go into the other number. So 600 divided by 300 is 2.

What is the solution to this linear system?

Answers

The answer is (-1,7)

I got this answer because to solve a system, you find the point on the graph where both lines intersect. And that point in this graph is (-1,7). Hope this helps!:)))))
Other Questions
The midpoint of AB is M(1,-2). If the coordinates of A are (-3, -8), what arethe coordinates of B? A business student is preparing for the Graduate Management Admission Test (GMAT) so he can get into a good graduate business school. He knows that any money he spends on a tutor will be worthwhile if he can improve his scores. He's heard that there is a great tutor in his local community but has no idea who she is. Marketing does not occur in this situation because What are the three outputs a result of photosynthesis? Will give brilliant ( when I can) 1. ____ which is also known as sugar 2. _____ _______ 3. ______= O2Of the 3 outputs, _____ is the actual food for the plant 9x + 5 - 3x = 5x - 14 Write the names of the reactants and products of photosynthesis and chemosynthesis.+++Light EnergyChemical Energy1. Describe how photosynthesis and chemosynthesis differ in terms of how energy is converted, what factor is not a characteristics of a mineral What were precomputed tables and why were they necessary? What is the volume of a book when the length is 42 cm, the width is 12 cm and the height is 4 cm?What is the mass of a rock if the volume is 54 ml and the density is 3.2 g/ml?What is the density of a block of wood that has a mass of 38 g and a volume of 19 cm3?What is the mass of a liquid that has a density of 2.4 g/ml and a volume of 32 ml?What is the volume of gold with the mass of 1500 g if the density of gold is 19.32 g/ml?What is the density of a box if the mass is 160 g and the volume is 12 ml?What is the volume of a sharks tooth when you start with 75 ml of water in a graduated cylinder and when the sharks tooth is placed in the water, the water level rises to 90 ml? (use water displacement) The mass of a pencil is 8 g and the density is 1.24 g/ cm3. What is the volume of the pencil? What indicates that an atomhas no overall charge? plz help if you can.. 245:47345678910Vince weighs 160 pounds, and his friend Nadir weighs 140 pounds. Nadir calculated that his weight on anotherplanet would be about 56 lb. Approximately what would Vince weigh on the other planet?40 lb64 lb76 lb90 lbSubmitNeyt 30 divided by 1.25 with work how can you identify a consumer? Act that are awarded at home Which of the following is not true about templates?It incorporates formatting elements.They slow down the process of creating presentations.It incoprorates layout options.It may include content that can be modified. !!20 POINTS!!Find AC 4= 6/a x +5 solve for x 6) The rectangle below has a perimeter of 46. Find its area: WILL GIVE BRAINLIEST Spain wanted to protect the border of New Spain so itsent an infantry into Texasb. established missionsC. waged war against Franced. sent an expedition into Texas how do u say how are u in german?